CME4PAC

Online CME for Physician Assistants

Menu

Skip to content
  • Home
  • Membership Benefits
  • My Account
  • About Us
  • Log In
  • Contact Us
  • Ortho on the Go

October 2017 NHSPA CME Quiz

Time limit: 0

Quiz-summary

0 of 8 questions completed

Questions:

  1. 1
  2. 2
  3. 3
  4. 4
  5. 5
  6. 6
  7. 7
  8. 8

Information

October 2017 NHSPA CME Quiz
At the conclusion of this quiz PAs should be able to recognize tick borne diseases, be able to differentiates causes of electrolyte imbalances, know risk factors for breast cancer, know the most common causes of Bell’s Palsy, know the pathological findings with ulcerative colitis, know how to treat acute pancreatitis, know the Kellgren-Lawrence (KL) grading system for grading osteoarthritis, and know the diagnostic work-up of a Morton’s Neuroma.

You have already completed the quiz before. Hence you can not start it again.

Quiz is loading...

You must sign in or sign up to start the quiz.

You have to finish following quiz, to start this quiz:

Results

0 of 8 questions answered correctly

Your time:

Time has elapsed

You have reached 0 of 0 points, (0)

Average score
 
 
Your score
 
 

Categories

  1. Not categorized 0%
  • You are required to complete the post-quiz evaluation to receive your CME credit and certificate. Please complete the post-quiz evaluation here. You will receive your CME certificate via email at the end of this CME calendar year. Please note that this CME activity was sponsored by the New Hampshire Society of Physician Assistants (NHSPA). When logging credit into your NCCPA account, the activity tittle will be “October 2017 NHSPA CME Quiz”.

  1. 1
  2. 2
  3. 3
  4. 4
  5. 5
  6. 6
  7. 7
  8. 8
  1. Answered
  2. Review
  1. Question 1 of 8
    1. Question

    A 56 year old woman with a past medical history of fibrocystic breast disease diagnosed in her 20s, one child delivered by cesarean section at age 33 (G1P1), and hypertension presents to her gynecologist with a lump in her right breast. She has a family history present for a maternal grandmother diagnosed with breast cancer at age 68 and a father with lung cancer. She denies any nipple discharge or skin changes. How many risk factors for breast cancer does this patient have?

    Correct

    Correct Answer: (C) The patient has four risk factors present for breast cancer. She has a previous history of a benign breast condition (fibrocystic breast disease), she had her first child after the age of 30, she has a family history of breast cancer and she is 56 years old.
    Incorrect Answers: (A) There is more than one risk factor.
    (B) There are more than three risk factors.
    (D) There are not five risk factors; hypertension is not a risk factor for breast cancer.

    Discussion:
    Invasive ductal carcinoma also known as infiltrating ductal carcinoma is the most common type of invasive breast cancer. It accounts for 70-80% of invasive breast lesions. (1) Although it can be seen in women of any age it is most commonly seen in women over the age of 55 (3). IDC is a cancer that began growing in the milk ducts of the breast and has invaded the fatty tissue of the breast as well. Cytological features of the involved cells range from bland to highly malignant. The malignant cells prompt a fibrous response as they infiltrate the breast tissue which is what causes a clinically palpable mass, the density that can be seen radiologically, and solid sonographic characteristics of classic invasive carcinomas (1). It is not clear what actually causes the abnormal cell growth but some risk factors have been identified. These risk factors include increasing age, personal history of benign breast disease, family history of breast cancer, first pregnancy after the age of 30 or nulliparity, history of DES exposure, hormone replacement therapy, obesity and genetic factors such as the BRCA1 or BRCA2 gene (2). Clinical features of invasive ductal carcinoma usually involve a mass or lesion that is a hard, immovable, lesion with irregular borders. However, the clinical features cannot consistently distinguish a benign tumor from a malignant one. If the cancer is locally advanced the signs can include axillary adenopathy or skin findings such as erythema, or dimpling of the overlying skin known as peau d’orange. If the cancer has metastases the symptoms can greatly vary depending on what organs are involved. The most common site of metastatic involvement is the bone, followed by the liver and lungs. Signs can include back or leg pain, abdominal pain, nausea, jaundice, and shortness of breath or cough. Most commonly however, patients present due to an abnormal mammogram finding (4).
    Classic findings of breast cancer on mammogram include the presence of a soft tissue mass or density as well as clustered microcalcifications invading to surrounding tissues. Breast ultrasound is often used in younger women due to the density of their breasts as well as to distinguish a benign versus malignant lesion. Sonographic features of breast malignancy include the presence of “spiculation, hypoechogenicity, microlobulation, internal calcifications, shadowing, a lesion taller than it is wide and angular margins” (4). Magnetic resonance imaging (MRI) is typically used to screen women at high risk for breast cancer as well as to further distinguish cancers seem on mammogram or ultrasound. Even though nearly all invasive breast cancers will enhance on gadolinium contrast-enhanced MRI, it is still not specific enough to remove the need for biopsy. Normally, once a lesion or mass is seen on mammogram, ultrasound, and/or MRI and it is suspicious for malignancy, it will be biopsied either using FNA or a core needle biopsy (4).
    As far as management and treatment goes, it really depends on the stage and grade of the cancer as well as the patient characteristics. In most cases, treatment will begin with surgery not only to remove the cancer but also to determine if any lymph nodes are involved. Possible surgical options include lumpectomy or different variations of mastectomy including partial, total and radical. Sentinel lymph node and axillary dissections are also usually done during the procedure to help stage the cancer and see the extent to which it has spread to the lymphatic system. Radiation and chemotherapy are also treatment options again, depending on the stage of the cancer and the extent to which it has metastasized (3).
    References:
    Bleiweiss IJ. Pathology of Breast Cancer. Pathology of breast cancer. https://www.uptodate.com/contents/pathology-of-breast-cancer?source=search_result&search=invasive ductal carcinoma&selectedTitle=1~150. Published June 17, 2016. Accessed August 1, 2017.
    Ductal carcinoma in situ (DCIS). Mayo Clinic. http://www.mayoclinic.org/diseases-conditions/dcis/basics/definition/con-20031842. Published June 14, 2014. Accessed August 1, 2017.
    Esserman L J, Joe BN. Clinical features, diagnosis, and staging of newly diagnosed breast cancer. Clinical features, diagnosis, and staging of newly diagnosed breast cancer. https://www.uptodate.com/contents/clinical-features-diagnosis-and-staging-of-newly-diagnosed-breast-cancer?source=search_result&search=invasive infiltrating ductal breast cancer&selectedTitle=1~150#H28333270. Published February 2, 2017. Accessed August 1, 2017.
    IDC – Invasive Ductal Carcinoma. Breastcancer.org. http://www.breastcancer.org/symptoms/types/idc. Accessed August 1, 2017.

    Incorrect

    Correct Answer: (C) The patient has four risk factors present for breast cancer. She has a previous history of a benign breast condition (fibrocystic breast disease), she had her first child after the age of 30, she has a family history of breast cancer and she is 56 years old.
    Incorrect Answers: (A) There is more than one risk factor.
    (B) There are more than three risk factors.
    (D) There are not five risk factors; hypertension is not a risk factor for breast cancer.

    Discussion:
    Invasive ductal carcinoma also known as infiltrating ductal carcinoma is the most common type of invasive breast cancer. It accounts for 70-80% of invasive breast lesions. (1) Although it can be seen in women of any age it is most commonly seen in women over the age of 55 (3). IDC is a cancer that began growing in the milk ducts of the breast and has invaded the fatty tissue of the breast as well. Cytological features of the involved cells range from bland to highly malignant. The malignant cells prompt a fibrous response as they infiltrate the breast tissue which is what causes a clinically palpable mass, the density that can be seen radiologically, and solid sonographic characteristics of classic invasive carcinomas (1). It is not clear what actually causes the abnormal cell growth but some risk factors have been identified. These risk factors include increasing age, personal history of benign breast disease, family history of breast cancer, first pregnancy after the age of 30 or nulliparity, history of DES exposure, hormone replacement therapy, obesity and genetic factors such as the BRCA1 or BRCA2 gene (2). Clinical features of invasive ductal carcinoma usually involve a mass or lesion that is a hard, immovable, lesion with irregular borders. However, the clinical features cannot consistently distinguish a benign tumor from a malignant one. If the cancer is locally advanced the signs can include axillary adenopathy or skin findings such as erythema, or dimpling of the overlying skin known as peau d’orange. If the cancer has metastases the symptoms can greatly vary depending on what organs are involved. The most common site of metastatic involvement is the bone, followed by the liver and lungs. Signs can include back or leg pain, abdominal pain, nausea, jaundice, and shortness of breath or cough. Most commonly however, patients present due to an abnormal mammogram finding (4).
    Classic findings of breast cancer on mammogram include the presence of a soft tissue mass or density as well as clustered microcalcifications invading to surrounding tissues. Breast ultrasound is often used in younger women due to the density of their breasts as well as to distinguish a benign versus malignant lesion. Sonographic features of breast malignancy include the presence of “spiculation, hypoechogenicity, microlobulation, internal calcifications, shadowing, a lesion taller than it is wide and angular margins” (4). Magnetic resonance imaging (MRI) is typically used to screen women at high risk for breast cancer as well as to further distinguish cancers seem on mammogram or ultrasound. Even though nearly all invasive breast cancers will enhance on gadolinium contrast-enhanced MRI, it is still not specific enough to remove the need for biopsy. Normally, once a lesion or mass is seen on mammogram, ultrasound, and/or MRI and it is suspicious for malignancy, it will be biopsied either using FNA or a core needle biopsy (4).
    As far as management and treatment goes, it really depends on the stage and grade of the cancer as well as the patient characteristics. In most cases, treatment will begin with surgery not only to remove the cancer but also to determine if any lymph nodes are involved. Possible surgical options include lumpectomy or different variations of mastectomy including partial, total and radical. Sentinel lymph node and axillary dissections are also usually done during the procedure to help stage the cancer and see the extent to which it has spread to the lymphatic system. Radiation and chemotherapy are also treatment options again, depending on the stage of the cancer and the extent to which it has metastasized (3).
    References:
    Bleiweiss IJ. Pathology of Breast Cancer. Pathology of breast cancer. https://www.uptodate.com/contents/pathology-of-breast-cancer?source=search_result&search=invasive ductal carcinoma&selectedTitle=1~150. Published June 17, 2016. Accessed August 1, 2017.
    Ductal carcinoma in situ (DCIS). Mayo Clinic. http://www.mayoclinic.org/diseases-conditions/dcis/basics/definition/con-20031842. Published June 14, 2014. Accessed August 1, 2017.
    Esserman L J, Joe BN. Clinical features, diagnosis, and staging of newly diagnosed breast cancer. Clinical features, diagnosis, and staging of newly diagnosed breast cancer. https://www.uptodate.com/contents/clinical-features-diagnosis-and-staging-of-newly-diagnosed-breast-cancer?source=search_result&search=invasive infiltrating ductal breast cancer&selectedTitle=1~150#H28333270. Published February 2, 2017. Accessed August 1, 2017.
    IDC – Invasive Ductal Carcinoma. Breastcancer.org. http://www.breastcancer.org/symptoms/types/idc. Accessed August 1, 2017.

  2. Question 2 of 8
    2. Question

    A 17 year-old male is referred to your office by his PCP after reporting new onset of frank blood in stools after 6 weeks of intermittent diarrhea. CBC, LFT’s, BMP, and stool studies were all initiated and only Signiant lab result of note was ESR elevated at 41. Decision was made at your office for a complete GI workup complete with an upper and lower endoscopy. Endoscopy of the colon with multiple biopsy retrievals concluded the diagnosis of Ulcerative Colitis and the patient was started on Lialda (mesalamine) 3.6mg daily. Which of the following are you most likely to read on the biopsy pathology results confirming the diagnosis of Ulcerative Colitis?

    Correct

    Answer is C.

    Discussion:
    Ulcerative Colitis is an inflammatory disease that effects the colon. Ulcerative Colitis is a subcategory of Inflammatory Bowel Disease and often has a similar presentation to its counterpart, Crohn’s Disease. Ulcerative Colitis is an inflammatory process which is believed to be idiopathic, although there are many factors that are thought to influence the development of ulcerative colitis, there is no root underlying cause to the disease progression. Some factors that are believed to influence the development of ulcerative colitis are heredity, triggers to the immune system and environmental factors such as viruses 1. All of these factors are suspected to have an impact on the body’s immune system and inflammation response in the large intestine.
    Ulcerative colitis is defined as an inflammatory condition characterized by relapsing and remitting episodes of inflammation limited to the mucosal layer of the colon. While Crohn’s disease is characterized by involvement of any portion of the GI tract, Ulcerative colitis involves the colon, starting in the rectum and spreading proximally. Ulcerative Colitis commonly presents in the younger populations commonly between the ages of 15 and 35, but has also be documented in children younger than 15 years of age. According to the Crohn’s and Ulcerative Colitis Foundation over 500,000 Americans are living with Ulcerative Colitis.
    In patients diagnosed with Ulcerative Colitis, pathological changes to the normal mucosal layer of the colon include inflammatory and morphological changes. While the underlying mechanism of the direct cause of these mucosal changes are unknown, there are multiple hypotheses that result in these changes. One hypothesis is that there is an autoimmune response to a mucosal antigen which results in morphological changes to the mucosal layer of the large intestine causing inflammation. Another hypothesis explores the possibly of a dysregulation of the immune function to fight of normal bacterial and flora found in the large intestine. Finally, there is also a hypothesis that explores the immune system dysregulation to fight off and destroy viruses and in result continuous inflammation of the large intestine results. The inflammation seen in Ulcerative Colitis affects the mucosa and sometimes the submucosa, and often a clear border is seen between normal and affected tissue. Early in the disease and with mild presentation, the mucous membrane is erythematous, with loss of the normal vascular pattern and depicts a friable pattern and often with scattered hemorrhagic areas. In severe presentations morphologic changes include scattered large mucosal ulcers.
    Many patients with Ulcerative colitis may present in early stages with minor and vague abdominal complaints; such as diarrhea, gas, bloating, crampy pain and fatigue. Important history to take on a patient with these complaints and suspicion of UC are family history, recent travel, medication uses and previous radiation exposures. In more active times of disease, patients with complain of bloody stools, bouts of diarrhea, rectal urgency, fatigue and possibly fever. On exam it is important to look for signs of dehydration with multiple day history of diarrhea, such as orthostatic readings or urine analysis. Physical exam may yield unremarkable findings or lower quadrant abdominal pain to palpation. During a severe flare, patients may present with more systemic symptoms such as fever and night sweats and can have arthritis which is a frequent extra-intestinal finding.
    Patients who present with a high suspicion of these complaints should have blood work and stool studies completed. Routine blood work such as a CBC, BMP, LFTs, as well as inflammatory markers CRP and ESR should be ordered. Stool studies such as C.Diff, ova and parasite, fecal calprotectin and lactoferrin should also be considered to rule out other infectious causes2. Blood work may yield signs of anemias or elevated inflammatory markers. Low levels of Albumin and electrolyte abnormalities are also commonly seen in patients with ulcerative colitis2. While blood work may help strengthen the suspicion of UC, these markers are not diagnostic for UC. Abdominal X-Ray may be preformed in patients with vague complaints and may show evidence of mucosal thickening or “thumbprinting”2. The definitive diagnosis must be obtained through biopsies through an endoscopy of the colon. Common findings from an endoscopy of the colon are marked erythema and loss of vascular markings of the colon mucosa, as well friable tissue and bloody exudate. Inflammation most always involved the rectum in diagnosis of ulcerative colitis and is commonly an initial presentation on endoscopy. Other common and classic findings on endoscopy of the colon in a patient with UC are pseudo polyps and crypt abscesses, which represent an acute flare of disease3.
    Once definitive diagnosis has been made many patients will undergo therapy to control the flare of the disease. First line treatment for UC is a topical 5-aminosalicyclic (5-ASA) medication that comes in many different forms such as oral, enemas, and suppositories. Commonly patients will prefer oral medication but for those with more severe symptoms or recurrent flare, other combination routes can be given as well. According to UpToDate guidelines maintenance therapy is not recommended to patients who have been diagnosed with a first episode or have mild disease and have responded will to 5-ASA treatment2. Patients who do not have good results and experience many episodes of recurrence on the 5-ASA can also be placed on oral corticosteroids with a taper for symptoms. Patients are also encouraged to watch their diets and avoid foods that may be irritating to the stomach. Ulcerative colitis is a lifelong disorder and the goal is untimely reemission of symptoms and preventing flares of disease.

    References:

    1. Crohn’s Disease Diagnosis & Testing. Crohn’s and Colitis’. https://www.crohnsandcolitis.com/crohns/testing-for-crohns. Accessed August 16, 2017.

    2. Peppercorn, MD. Clinical manifestations, diagnosis, and prognosis of ulcerative colitis in adults. Post TW, ed. UpToDate. Waltham, MA: UpToDate Inc. http://www.uptodate.com

    3. Ulcerative Colitis – Gastrointestinal Disorders. Merck Manuals Professional Edition. http://www.merckmanuals.com/professional/gastrointestinal-disorders/inflammatory-bowel-disease-ibd/ulcerative-colitis. Accessed August 16, 2017.

    4. Ulcerative colitis. Ulcerative colitis (Idiopathic Proctocolitis) – treatment and medications. http://www.nmihi.com/c/ulcerative-colitis.htm. Accessed August 16, 2017.

    5. Hendrickson BA, Gokhale R, Cho JH. Clinical Aspects and Pathophysiology of Inflammatory Bowel Disease. Clinical Microbiology Reviews. 2002;15(1):79-94. doi:10.1128/CMR.15.1.79-94.2002.

    Incorrect

    Answer is C.

    Discussion:
    Ulcerative Colitis is an inflammatory disease that effects the colon. Ulcerative Colitis is a subcategory of Inflammatory Bowel Disease and often has a similar presentation to its counterpart, Crohn’s Disease. Ulcerative Colitis is an inflammatory process which is believed to be idiopathic, although there are many factors that are thought to influence the development of ulcerative colitis, there is no root underlying cause to the disease progression. Some factors that are believed to influence the development of ulcerative colitis are heredity, triggers to the immune system and environmental factors such as viruses 1. All of these factors are suspected to have an impact on the body’s immune system and inflammation response in the large intestine.
    Ulcerative colitis is defined as an inflammatory condition characterized by relapsing and remitting episodes of inflammation limited to the mucosal layer of the colon. While Crohn’s disease is characterized by involvement of any portion of the GI tract, Ulcerative colitis involves the colon, starting in the rectum and spreading proximally. Ulcerative Colitis commonly presents in the younger populations commonly between the ages of 15 and 35, but has also be documented in children younger than 15 years of age. According to the Crohn’s and Ulcerative Colitis Foundation over 500,000 Americans are living with Ulcerative Colitis.
    In patients diagnosed with Ulcerative Colitis, pathological changes to the normal mucosal layer of the colon include inflammatory and morphological changes. While the underlying mechanism of the direct cause of these mucosal changes are unknown, there are multiple hypotheses that result in these changes. One hypothesis is that there is an autoimmune response to a mucosal antigen which results in morphological changes to the mucosal layer of the large intestine causing inflammation. Another hypothesis explores the possibly of a dysregulation of the immune function to fight of normal bacterial and flora found in the large intestine. Finally, there is also a hypothesis that explores the immune system dysregulation to fight off and destroy viruses and in result continuous inflammation of the large intestine results. The inflammation seen in Ulcerative Colitis affects the mucosa and sometimes the submucosa, and often a clear border is seen between normal and affected tissue. Early in the disease and with mild presentation, the mucous membrane is erythematous, with loss of the normal vascular pattern and depicts a friable pattern and often with scattered hemorrhagic areas. In severe presentations morphologic changes include scattered large mucosal ulcers.
    Many patients with Ulcerative colitis may present in early stages with minor and vague abdominal complaints; such as diarrhea, gas, bloating, crampy pain and fatigue. Important history to take on a patient with these complaints and suspicion of UC are family history, recent travel, medication uses and previous radiation exposures. In more active times of disease, patients with complain of bloody stools, bouts of diarrhea, rectal urgency, fatigue and possibly fever. On exam it is important to look for signs of dehydration with multiple day history of diarrhea, such as orthostatic readings or urine analysis. Physical exam may yield unremarkable findings or lower quadrant abdominal pain to palpation. During a severe flare, patients may present with more systemic symptoms such as fever and night sweats and can have arthritis which is a frequent extra-intestinal finding.
    Patients who present with a high suspicion of these complaints should have blood work and stool studies completed. Routine blood work such as a CBC, BMP, LFTs, as well as inflammatory markers CRP and ESR should be ordered. Stool studies such as C.Diff, ova and parasite, fecal calprotectin and lactoferrin should also be considered to rule out other infectious causes2. Blood work may yield signs of anemias or elevated inflammatory markers. Low levels of Albumin and electrolyte abnormalities are also commonly seen in patients with ulcerative colitis2. While blood work may help strengthen the suspicion of UC, these markers are not diagnostic for UC. Abdominal X-Ray may be preformed in patients with vague complaints and may show evidence of mucosal thickening or “thumbprinting”2. The definitive diagnosis must be obtained through biopsies through an endoscopy of the colon. Common findings from an endoscopy of the colon are marked erythema and loss of vascular markings of the colon mucosa, as well friable tissue and bloody exudate. Inflammation most always involved the rectum in diagnosis of ulcerative colitis and is commonly an initial presentation on endoscopy. Other common and classic findings on endoscopy of the colon in a patient with UC are pseudo polyps and crypt abscesses, which represent an acute flare of disease3.
    Once definitive diagnosis has been made many patients will undergo therapy to control the flare of the disease. First line treatment for UC is a topical 5-aminosalicyclic (5-ASA) medication that comes in many different forms such as oral, enemas, and suppositories. Commonly patients will prefer oral medication but for those with more severe symptoms or recurrent flare, other combination routes can be given as well. According to UpToDate guidelines maintenance therapy is not recommended to patients who have been diagnosed with a first episode or have mild disease and have responded will to 5-ASA treatment2. Patients who do not have good results and experience many episodes of recurrence on the 5-ASA can also be placed on oral corticosteroids with a taper for symptoms. Patients are also encouraged to watch their diets and avoid foods that may be irritating to the stomach. Ulcerative colitis is a lifelong disorder and the goal is untimely reemission of symptoms and preventing flares of disease.

    References:

    1. Crohn’s Disease Diagnosis & Testing. Crohn’s and Colitis’. https://www.crohnsandcolitis.com/crohns/testing-for-crohns. Accessed August 16, 2017.

    2. Peppercorn, MD. Clinical manifestations, diagnosis, and prognosis of ulcerative colitis in adults. Post TW, ed. UpToDate. Waltham, MA: UpToDate Inc. http://www.uptodate.com

    3. Ulcerative Colitis – Gastrointestinal Disorders. Merck Manuals Professional Edition. http://www.merckmanuals.com/professional/gastrointestinal-disorders/inflammatory-bowel-disease-ibd/ulcerative-colitis. Accessed August 16, 2017.

    4. Ulcerative colitis. Ulcerative colitis (Idiopathic Proctocolitis) – treatment and medications. http://www.nmihi.com/c/ulcerative-colitis.htm. Accessed August 16, 2017.

    5. Hendrickson BA, Gokhale R, Cho JH. Clinical Aspects and Pathophysiology of Inflammatory Bowel Disease. Clinical Microbiology Reviews. 2002;15(1):79-94. doi:10.1128/CMR.15.1.79-94.2002.

  3. Question 3 of 8
    3. Question

    A 44 year old male presents to the emergency department complaining of abdominal pain. He states he has never felt anything like this in the past. He rates his pain as 8/10 and describes it as constant pain in his upper abdomen that radiates to his back. He says that he went out and had “a few too many drinks last night” but his stomach felt okay when he went to bed. At some point during the night his pain came on suddenly and hasn’t gone away. Labs were done on this patient and his serum amylase was 458U/L (normal value: 23-85U/L), lipase was 3948U/L (normal value: 0-160U/L), and triglycerides were 1328 (normal value: <159mg/dL). What is the best treatment option for this patient?

    Correct

    In the United States, one of the most frequent GI causes for hospitalization is pancreatitis1. Pancreatitis has many etiologies, and may be acute or chronic. The most common cause of acute pancreatitis is gallstones, followed by alcohol use. Smoking cigarettes is not a known cause on its own, but when combined with alcohol use the effect will be synergistic and the likelihood of getting acute pancreatitis is heightened1. The third most common cause of pancreatitis is hypertriglyceridemia2. Hypertriglyceridemia is determined when fasting triglycerides are greater than 150 mg/dL. Mild hypertriglyceridemia is 150-199 mg/dL, moderate is 200-999mg/dL, severe is 1000-1999mg/dL and very severe hypertriglyceridemia is greater than 2000 mg/dL. When triglyceride levels are above 500mg/dL there is a progressive risk for acute pancreatitis. Levels above 1000 mg/dL increases the risk of acute pancreatitis even further2. Drinking alcohol can also contribute to hypertriglyceridemia because it increases serum triglyceride concentrations in a “dose dependent” manner. Having hypertriglyceride-induced pancreatitis has a worse outcome than other types of pancreatitis2.
    When a person has hypertriglyceridemia, they have an increased concentration of chylomicrons3. Chylomicrons are large lipoproteins with very low density that are formed 1-3 hours after eating, and are typically cleared within 8 hours. They are a derivative of dietary fat and their job is to carry triglycerides throughout the body4. When the triglyceride level is above 1000mg/dL, the concentration of chylomicrons is always increased3. Due to their large, low density nature, they may obstruct capillaries leading to local ischemia and acidemia. This damage can expose triglycerides to pancreatic lipase which will further degrade the triglycerides to free fatty acids causing cytotoxic injury. This local injury will increase the inflammatory mediators and free radicals in the body, and eventually manifest into acute pancreatitis3.
    When working a patient up for pancreatitis it is important to ask about their history and family history of gallstones since that is the number one cause of pancreatitis. It is also important to ask about alcohol use and diet since pancreatitis is also commonly caused from excess alcohol use, and from hypertriglyceridemia. If the patient smokes cigarettes as well as drinking alcohol they are even more likely to have an episode of acute pancreatitis. The patient used in this case had a history of alcohol abuse, cigarette smoking, dyslipidemia, and coronary artery disease which are all pertinent to pancreatitis. On physical exam, his epigastric region was extremely tender to palpation, with pain rising to 10/10. He described his pain as a sudden onset, chronic pain in his upper abdomen radiating to his back, which is a very typical presentation of acute pancreatitis. His labs showed extremely high lipase at 3948U/L, and his liver enzymes were also elevated. On first draw his triglycerides and cholesterol/LDL were unable to detect due to specimen integrity, but they were estimated to be extremely elevated based on appearance. The tube showed a thick lactescent serum indicative of high triglycerides. Between that and his onset of acute pancreatitis it was estimated that his triglycerides were over 1000mg/dL. The exact level of triglycerides necessary to induce acute pancreatitis is unknown, but it is believed to happen at a level over 1000mg/dL5.
    When it comes to acute pancreatitis caused by hypertriglyceridemia, there are a few treatment options to choose from. The first line choice is apheresis, however this is not always available. Apheresis is a procedure that removes whole blood from the patient and runs it through a machine that will separate the blood into individual components. This process is allowing a specific component to be removed, and then the remaining components will be returned back into the blood stream of the patient. This process typically takes about two hours6. This is a great option for hypertriglyceridemia-induced pancreatitis because it can remove the unwanted lipids from the blood.
    Another option to decrease triglycerides is using an insulin drip. This method is preferred over apheresis if the patient has a glucose level above 500mg/dL. It is also used when apheresis is not available2. Hypertriglyceridemia is caused when there is a direct lipoprotein lipase deficiency or a decrease in lipoprotein lipase activity3. Lipoprotein lipase is an enzyme that accelerates chylomicron and VLDL metabolism to glycerol and free fatty acids2. Insulin will enhance lipoprotein lipase activity and help reduce triglyceride levels3. The protocol for using insulin to reduce triglycerides is to set up an IV drip of 0.1 – 0.3 units/kg/hour. If a patient is not hyperglycemic and an insulin drip is started, IV glucose supplementation like D5W must be added to prevent hypoglycemia. The insulin drip should be discontinued when triglycerides return to a level below 500mg/dL2.
    In the case study question of the best treatment option for our patient, “Keep this patient NPO, give IV fluids, insulin drip at 0.2 units/kg/hour and pain management” is the correct answer. The first answer is incorrect, because the role of heparin in reducing triglycerides is controversial. It has been used with some success, but the problem with using heparin is that after the initial benefit of the drug, there is a rebound effect that will increase hepatic degradation of lipoprotein lipase which further decreases plasma stores of lipoprotein lipase and ultimately results in an increase in chylomicrons2. High levels of chylomicrons contributed to the cause of acute pancreatitis in the first place, so it is best to avoid this. The second answer, “Keep the patient NPO and give pain management,” is incorrect because although you want to keep the patient NPO and relieve their pain, this is not doing enough for the patient. It is important to resuscitate with IV fluids. The final answer that includes NPO, pain management, and IV fluids is still incorrect for this particular patient. Although most cases of pancreatitis are treated by doing exactly this combination, our patient has triglycerides greater than 1000mg/dL. Without getting his triglyceride levels to decrease his pancreatitis will not resolve. The best answer is adding the insulin IV drip because that will be able to get his triglyceride level down. That in combination with NPO, IV fluids, and pain management is the treatment of choice to obtain the best possible outcome for this patient.

    References
    1. Yadav D, Lowenfels A. The Epidemiology of Pancreatitis and Pancreatic Cancer. National Center for Biotechnology Information. 2013. Available at: https://www.ncbi.nlm.nih.gov/pmc/articles/PMC3662544/. Accessed August 5, 2017.
    2. Gelrud A, Whitcomb D. Hypertriglyceridemia-induced acute pancreatitis. UpToDate. 2015. Available at: https://www.uptodate.com/contents/hypertriglyceridemia-induced-acute-pancreatitis. Accessed August 2, 2017.
    3. Frenkel D. Why Does Hypertriglyceridemia Lead to Pancreatitis? | Clinical Correlations. Clinical Correlations. 2007. Available at: http://www.clinicalcorrelations.org/?p=478. Accessed August 2, 2017.
    4. Engelking L. Chylomicron – an overview | ScienceDirect Topics. Sciencedirectcom. 2015. Available at: http://www.sciencedirect.com/topics/neuroscience/chylomicron. Accessed August 5, 2017.
    5. Tamás. Baranyai, ViktAória. Terzin, ágota. Vajda, Tibor. Wittmann & László. Czakó (2012) Hypertriglyceridemia causes more severe course of acute pancreatitis, Clinical Lipidology, 7:6, 731-736. Accessed August 5, 2017
    6. Stoppler M. Apheresis: Facts on Platelet Donation Procedures. MedicineNet. 2016. Available at: http://www.medicinenet.com/hemapheresis/article.htm. Accessed August 5, 2017.

    Incorrect

    In the United States, one of the most frequent GI causes for hospitalization is pancreatitis1. Pancreatitis has many etiologies, and may be acute or chronic. The most common cause of acute pancreatitis is gallstones, followed by alcohol use. Smoking cigarettes is not a known cause on its own, but when combined with alcohol use the effect will be synergistic and the likelihood of getting acute pancreatitis is heightened1. The third most common cause of pancreatitis is hypertriglyceridemia2. Hypertriglyceridemia is determined when fasting triglycerides are greater than 150 mg/dL. Mild hypertriglyceridemia is 150-199 mg/dL, moderate is 200-999mg/dL, severe is 1000-1999mg/dL and very severe hypertriglyceridemia is greater than 2000 mg/dL. When triglyceride levels are above 500mg/dL there is a progressive risk for acute pancreatitis. Levels above 1000 mg/dL increases the risk of acute pancreatitis even further2. Drinking alcohol can also contribute to hypertriglyceridemia because it increases serum triglyceride concentrations in a “dose dependent” manner. Having hypertriglyceride-induced pancreatitis has a worse outcome than other types of pancreatitis2.
    When a person has hypertriglyceridemia, they have an increased concentration of chylomicrons3. Chylomicrons are large lipoproteins with very low density that are formed 1-3 hours after eating, and are typically cleared within 8 hours. They are a derivative of dietary fat and their job is to carry triglycerides throughout the body4. When the triglyceride level is above 1000mg/dL, the concentration of chylomicrons is always increased3. Due to their large, low density nature, they may obstruct capillaries leading to local ischemia and acidemia. This damage can expose triglycerides to pancreatic lipase which will further degrade the triglycerides to free fatty acids causing cytotoxic injury. This local injury will increase the inflammatory mediators and free radicals in the body, and eventually manifest into acute pancreatitis3.
    When working a patient up for pancreatitis it is important to ask about their history and family history of gallstones since that is the number one cause of pancreatitis. It is also important to ask about alcohol use and diet since pancreatitis is also commonly caused from excess alcohol use, and from hypertriglyceridemia. If the patient smokes cigarettes as well as drinking alcohol they are even more likely to have an episode of acute pancreatitis. The patient used in this case had a history of alcohol abuse, cigarette smoking, dyslipidemia, and coronary artery disease which are all pertinent to pancreatitis. On physical exam, his epigastric region was extremely tender to palpation, with pain rising to 10/10. He described his pain as a sudden onset, chronic pain in his upper abdomen radiating to his back, which is a very typical presentation of acute pancreatitis. His labs showed extremely high lipase at 3948U/L, and his liver enzymes were also elevated. On first draw his triglycerides and cholesterol/LDL were unable to detect due to specimen integrity, but they were estimated to be extremely elevated based on appearance. The tube showed a thick lactescent serum indicative of high triglycerides. Between that and his onset of acute pancreatitis it was estimated that his triglycerides were over 1000mg/dL. The exact level of triglycerides necessary to induce acute pancreatitis is unknown, but it is believed to happen at a level over 1000mg/dL5.
    When it comes to acute pancreatitis caused by hypertriglyceridemia, there are a few treatment options to choose from. The first line choice is apheresis, however this is not always available. Apheresis is a procedure that removes whole blood from the patient and runs it through a machine that will separate the blood into individual components. This process is allowing a specific component to be removed, and then the remaining components will be returned back into the blood stream of the patient. This process typically takes about two hours6. This is a great option for hypertriglyceridemia-induced pancreatitis because it can remove the unwanted lipids from the blood.
    Another option to decrease triglycerides is using an insulin drip. This method is preferred over apheresis if the patient has a glucose level above 500mg/dL. It is also used when apheresis is not available2. Hypertriglyceridemia is caused when there is a direct lipoprotein lipase deficiency or a decrease in lipoprotein lipase activity3. Lipoprotein lipase is an enzyme that accelerates chylomicron and VLDL metabolism to glycerol and free fatty acids2. Insulin will enhance lipoprotein lipase activity and help reduce triglyceride levels3. The protocol for using insulin to reduce triglycerides is to set up an IV drip of 0.1 – 0.3 units/kg/hour. If a patient is not hyperglycemic and an insulin drip is started, IV glucose supplementation like D5W must be added to prevent hypoglycemia. The insulin drip should be discontinued when triglycerides return to a level below 500mg/dL2.
    In the case study question of the best treatment option for our patient, “Keep this patient NPO, give IV fluids, insulin drip at 0.2 units/kg/hour and pain management” is the correct answer. The first answer is incorrect, because the role of heparin in reducing triglycerides is controversial. It has been used with some success, but the problem with using heparin is that after the initial benefit of the drug, there is a rebound effect that will increase hepatic degradation of lipoprotein lipase which further decreases plasma stores of lipoprotein lipase and ultimately results in an increase in chylomicrons2. High levels of chylomicrons contributed to the cause of acute pancreatitis in the first place, so it is best to avoid this. The second answer, “Keep the patient NPO and give pain management,” is incorrect because although you want to keep the patient NPO and relieve their pain, this is not doing enough for the patient. It is important to resuscitate with IV fluids. The final answer that includes NPO, pain management, and IV fluids is still incorrect for this particular patient. Although most cases of pancreatitis are treated by doing exactly this combination, our patient has triglycerides greater than 1000mg/dL. Without getting his triglyceride levels to decrease his pancreatitis will not resolve. The best answer is adding the insulin IV drip because that will be able to get his triglyceride level down. That in combination with NPO, IV fluids, and pain management is the treatment of choice to obtain the best possible outcome for this patient.

    References
    1. Yadav D, Lowenfels A. The Epidemiology of Pancreatitis and Pancreatic Cancer. National Center for Biotechnology Information. 2013. Available at: https://www.ncbi.nlm.nih.gov/pmc/articles/PMC3662544/. Accessed August 5, 2017.
    2. Gelrud A, Whitcomb D. Hypertriglyceridemia-induced acute pancreatitis. UpToDate. 2015. Available at: https://www.uptodate.com/contents/hypertriglyceridemia-induced-acute-pancreatitis. Accessed August 2, 2017.
    3. Frenkel D. Why Does Hypertriglyceridemia Lead to Pancreatitis? | Clinical Correlations. Clinical Correlations. 2007. Available at: http://www.clinicalcorrelations.org/?p=478. Accessed August 2, 2017.
    4. Engelking L. Chylomicron – an overview | ScienceDirect Topics. Sciencedirectcom. 2015. Available at: http://www.sciencedirect.com/topics/neuroscience/chylomicron. Accessed August 5, 2017.
    5. Tamás. Baranyai, ViktAória. Terzin, ágota. Vajda, Tibor. Wittmann & László. Czakó (2012) Hypertriglyceridemia causes more severe course of acute pancreatitis, Clinical Lipidology, 7:6, 731-736. Accessed August 5, 2017
    6. Stoppler M. Apheresis: Facts on Platelet Donation Procedures. MedicineNet. 2016. Available at: http://www.medicinenet.com/hemapheresis/article.htm. Accessed August 5, 2017.

  4. Question 4 of 8
    4. Question

    An 81 year-old male presents to the emergency room with a 10-day history of fever, chills, night sweats, SOB, weakness and fatigue. He first noticed the symptoms after returning from vacation with his family on Cape Cod in Massachusetts. He presents with a 105.3° temperature, 90/50 BP, 115 bpm and 80% O2 sat. Physical exam reveals minor crackles in bilateral basilar lungs and a systolic ejection murmur heard best at the apex. Routine lab work reveals a hemolytic anemia, thrombocytopenia and a normal WBC count. What is the most likely cause of his current state?

    Correct

    Answer is C. Babesiosis is a tick-borne disease that is carried by the same vector as Lyme disease and found classically in the northeastern portion of the United States like Cape Cod, Massachusetts. Patients infected with Babesiosis usually present acutely with malaise, weakness, fatigue and fevers upwards of 105°F. Fever in these patients is typically followed by chills and night sweats. Hemolytic anemia and thrombocytopenia are common features of this disease and WBC counts can be low, normal or high. The Patient’s acute onset with flu-like symptoms and high-grade fever suggest an infection that effectively rules out Pulmonary Embolism. Although PE patients often present with Tachycardia, SOB, weakness, fatigue, low O2 Sat and possibly lung finding on physical examination, there is no connection between PE causing thrombocytopenia or hemolytic anemia. Endocarditis would cause high fevers, flu-like symptoms and a heart murmur, however, this patient has a history of valvular disease that would explain the murmur and Endocarditis has no correlation to hemolytic anemias or thrombocytopenia. Patients who develop endocarditis typically have a point of entry into the bloodstream such as IV drug use, recent dental surgery or open wounds which this patient did not have. Rocky Mountain spotted fever is another tick-borne illness that presents with similar symptoms and findings including thrombocytopenia and hemolytic anemia, however it is not commonly found in the northeastern United States and classically has a palmar rash associated with its presentation.

    Discussion:
    Babesiosis is an infectious parasitic protozoan that is predominantly found in the Northeastern and upper Midwestern regions of the United States.1 Other species are found in Europe; however, all species affect all ethnicities equally.1 The most common species of babesiosis is B. microti which is what our conversation will focus on. There are rarer species of Babesiosis such as B. divergens which can cause fulminant illness and are considered a medical emergency requiring blood exchange transfusion.1
    Babesiosis is carried by tick vectors most commonly Ixodes scapularis ticks (aka the Black-Legged Deer Ticks).1 Ixodes scapularis have 3 stages of development from larva to nymph to adult which all require what is called a “blood meal” to advance to the next stage.1 They obtain the parasite through multiple feedings from hosts and the protozoa multiplies in the gut wall of the tick before concentrating in the salivary glands.1 When the next feeding occurs, the parasite is transferred from the tick vector to a new host entering the bloodstream during the bite as mature trophozoites.1 The parasite infects RBCs directly and once in the cell, the trophozoites undergo asexual budding and replicated into 2 to 4 merozoites which leave the cell through direct lysis of the membrane.1 Alterations in RBC membranes can lead to excess fluid which manifests as pulmonary edema and possibly ARDS.1
    Pathophysiological manifestations of the disease can be seen as hemolytic anemia due to direct parasite-mediated lysis of red blood cells in the circulation. This leads to a low hematocrit, elevated lactate dehydrogenase, low hemoglobin and elevated total bilirubin.1 As a result of lysis, the RBC fragments can cause capillary blockage and microvascular stasis which can lead to serious CNS complications, liver, splenic or renal involvement.1
    Presentation with Babesiosis infections range from being asymptomatic to severely unstable and possibly fatal.1 The severity and course of the infection mainly depend on the species of Babesiosis, the patient’s immune status and the patient’s co-morbidities. Most patients typically become symptomatic with the first 6 weeks following a bite from an infected tick in an endemic area but more precisely it has been determined that the median interval for symptom onset is 37 days.1 Patients typically present with gradual onset of fatigue, weakness, malaise and fevers that can reach dangerously high fevers such as 105.6°F.1 Following the fever, patients will develop chills and night sweats.1 Less commonly symptoms will include headache, nausea, vomiting, diarrhea, anorexia, arthralgia, myalgias, neck stiffness, shortness of breath, dry cough, weight loss or dark urine.1
    Physical examination may reveal mild organomegaly, specifically splenomegaly, but lymphadenopathy is absent.1 As previously stated, hemolytic anemia is present in all patients but thrombocytopenia is also common.1 White blood cell counts have no significance on evaluation as they can range from decreased to normal to elevated.1
    Patients with severe babesiosis require hospitalization and are susceptible to potential complications include acute respiratory distress syndrome, disseminated intravascular coagulation, congestive heart failure, and renal failure.1 These usually occur when the anemia becomes sever and the parasitemia is high. In rare circumstances, high fever can cause a hyperadrenergic state leading to excessive orthostatic tachycardia, which in someone with preexisting heart disease, such as our individual, can precipitate lethal arrhythmias.1
    The definitive diagnosis of babesiosis should be made by examining blood with direct light microscopy and Wright-Giemsa Stain.2 Specifically, examination of thin blood smears is necessary as babesial parasites may be too small to be seen using thick blood smears. Babesiosis most commonly are seen as a pale blue ring with two red chromatic dots.2 You also can occasionally see the parasites arranged in tetrads known as “Maltese Cross.1”
    Polymerase chain reaction (PCR) is also useful for diagnosis especially in those with a low-level of parasitemia and can be used to detect any Babesia DNA levels in blood that show no parasites on blood smear.1 One downfall of PCR is that the DNA detected does not always equate to live parasites and can be detected in blood for up to 26 months post-infection.1
    Indirect immunofluorescence can be used in rare cases with high suspicion of babesiosis and negative blood smears and PCR.1 These cases are seen more in blood donors that have been implicated in transfusion-acquired disease or in asymptomatic patients who have cleared the disease without treatment but may have complications following the infection.1
    The diagnosis should be considered in patients who present with flu-like symptoms and who either live in or have traveled recently to endemic areas. Also, patients with symptom who have been previously diagnosed with Lyme disease, Anaplasmosis, Powassan or other tick-borne illnesses should also be suspected.
    Treatment should be initiated in any symptomatic patients with parasitemia confirmed via blood smear or PCR. If a patient is asymptomatic but has a confirmed diagnosis of babesiosis, treatment is not necessary until symptomatic. Asymptomatic individuals who have persistent infection for > 3months as confirmed by blood smear or PCR should also consider treatment.
    The most effective therapy is the use of antimicrobial agents, specifically atovaquone plus azithromycin or quinine plus clindamycin.3 Both options are equally efficacious but Clindamycin plus quinine is typically reserved for severe cases (requiring hospitalization) or those who are immunocompromised.1 The typical regimen is to take both drugs orally for 7 to 10 days.3 Atovaquone plus azithromycin is the preferred treatment as the combination is better tolerated with less adverse effects and those who cannot tolerate PO can be given IV quinidine and clindamycin.1 Atovaquone is better absorbed when taken with a fatty meal, however azithromycin from capsules have a 50% reduced absorption when taken with food, which is why immediate-release azithromycin suspension is preferred for efficacy.1
    The typical dosing for Atovaquone is 750mg orally every 12 hours and Azithromycin is 500mg/day orally on day 1 and 250mg/day from days 2-10.3 In the case of Clindamycin, 300-600mg is given IV every 6 hours or 600mg orally every 8 hours.3 Quinine is typically dosed at 650mg orally every 6 to 8 hours.3 The dosing is modified for different species of pathogen and for children.
    There are some serious side effects associated with these treatments. Serial ECG’s need to be monitored while on quinines as they can prolong QT intervals and precipitate Toursades de Pointes.3 It is for this reason, that we tailored our treatment toward avoiding this side effect in our patient who had already experienced ventricular arrhythmias during his work-up. Pregnant patients or those breastfeeding should not take quinine or atovaquone as it is a category C drug and leads to abnormal limb development and neurological complications.3
    If patients fail treatment with clindamycin plus quinine, there have been studies that have shown some success with azithromycin plus quinine.3 There also has been success with high doses of clindamycin plus azithromycin and doxycycline especially in immunocompromised patients with recurrent symptoms.1 Other antimalarial agents have been unsuccessful in treating babesiosis and should not be considered for alternative treatment. In cases with high parasitemia and severe hemolytic anemia, blood exchange transfusions should be considered.1

    References
    1. Gelfand, JA and Vannier, EG. Clinical manifestations, diagnosis, treatment, and prevention of babesiosis. https://www.uptodate.com/contents/clinical-manifestations-diagnosis-treatment-and-prevention-of-babesiosis?source=search_result&search=babesiosis&selectedTitle=1~7. Accessed 13 Aug. 2017
    2. CDC – DPDx – Malaria – Diagnostic Findings. https://www.cdc.gov/dpdx/malaria/dx.html. Accessed 13 Aug. 2017
    3. Babesiosis Treatment & Management: Approach Considerations, Pharmacologic Therapy, Exchange Transfusion and Supportive Care. http://emedicine.medscape.com/article/212605-treatment. Accessed 13 Aug. 2017

    Incorrect

    Answer is C. Babesiosis is a tick-borne disease that is carried by the same vector as Lyme disease and found classically in the northeastern portion of the United States like Cape Cod, Massachusetts. Patients infected with Babesiosis usually present acutely with malaise, weakness, fatigue and fevers upwards of 105°F. Fever in these patients is typically followed by chills and night sweats. Hemolytic anemia and thrombocytopenia are common features of this disease and WBC counts can be low, normal or high. The Patient’s acute onset with flu-like symptoms and high-grade fever suggest an infection that effectively rules out Pulmonary Embolism. Although PE patients often present with Tachycardia, SOB, weakness, fatigue, low O2 Sat and possibly lung finding on physical examination, there is no connection between PE causing thrombocytopenia or hemolytic anemia. Endocarditis would cause high fevers, flu-like symptoms and a heart murmur, however, this patient has a history of valvular disease that would explain the murmur and Endocarditis has no correlation to hemolytic anemias or thrombocytopenia. Patients who develop endocarditis typically have a point of entry into the bloodstream such as IV drug use, recent dental surgery or open wounds which this patient did not have. Rocky Mountain spotted fever is another tick-borne illness that presents with similar symptoms and findings including thrombocytopenia and hemolytic anemia, however it is not commonly found in the northeastern United States and classically has a palmar rash associated with its presentation.

    Discussion:
    Babesiosis is an infectious parasitic protozoan that is predominantly found in the Northeastern and upper Midwestern regions of the United States.1 Other species are found in Europe; however, all species affect all ethnicities equally.1 The most common species of babesiosis is B. microti which is what our conversation will focus on. There are rarer species of Babesiosis such as B. divergens which can cause fulminant illness and are considered a medical emergency requiring blood exchange transfusion.1
    Babesiosis is carried by tick vectors most commonly Ixodes scapularis ticks (aka the Black-Legged Deer Ticks).1 Ixodes scapularis have 3 stages of development from larva to nymph to adult which all require what is called a “blood meal” to advance to the next stage.1 They obtain the parasite through multiple feedings from hosts and the protozoa multiplies in the gut wall of the tick before concentrating in the salivary glands.1 When the next feeding occurs, the parasite is transferred from the tick vector to a new host entering the bloodstream during the bite as mature trophozoites.1 The parasite infects RBCs directly and once in the cell, the trophozoites undergo asexual budding and replicated into 2 to 4 merozoites which leave the cell through direct lysis of the membrane.1 Alterations in RBC membranes can lead to excess fluid which manifests as pulmonary edema and possibly ARDS.1
    Pathophysiological manifestations of the disease can be seen as hemolytic anemia due to direct parasite-mediated lysis of red blood cells in the circulation. This leads to a low hematocrit, elevated lactate dehydrogenase, low hemoglobin and elevated total bilirubin.1 As a result of lysis, the RBC fragments can cause capillary blockage and microvascular stasis which can lead to serious CNS complications, liver, splenic or renal involvement.1
    Presentation with Babesiosis infections range from being asymptomatic to severely unstable and possibly fatal.1 The severity and course of the infection mainly depend on the species of Babesiosis, the patient’s immune status and the patient’s co-morbidities. Most patients typically become symptomatic with the first 6 weeks following a bite from an infected tick in an endemic area but more precisely it has been determined that the median interval for symptom onset is 37 days.1 Patients typically present with gradual onset of fatigue, weakness, malaise and fevers that can reach dangerously high fevers such as 105.6°F.1 Following the fever, patients will develop chills and night sweats.1 Less commonly symptoms will include headache, nausea, vomiting, diarrhea, anorexia, arthralgia, myalgias, neck stiffness, shortness of breath, dry cough, weight loss or dark urine.1
    Physical examination may reveal mild organomegaly, specifically splenomegaly, but lymphadenopathy is absent.1 As previously stated, hemolytic anemia is present in all patients but thrombocytopenia is also common.1 White blood cell counts have no significance on evaluation as they can range from decreased to normal to elevated.1
    Patients with severe babesiosis require hospitalization and are susceptible to potential complications include acute respiratory distress syndrome, disseminated intravascular coagulation, congestive heart failure, and renal failure.1 These usually occur when the anemia becomes sever and the parasitemia is high. In rare circumstances, high fever can cause a hyperadrenergic state leading to excessive orthostatic tachycardia, which in someone with preexisting heart disease, such as our individual, can precipitate lethal arrhythmias.1
    The definitive diagnosis of babesiosis should be made by examining blood with direct light microscopy and Wright-Giemsa Stain.2 Specifically, examination of thin blood smears is necessary as babesial parasites may be too small to be seen using thick blood smears. Babesiosis most commonly are seen as a pale blue ring with two red chromatic dots.2 You also can occasionally see the parasites arranged in tetrads known as “Maltese Cross.1”
    Polymerase chain reaction (PCR) is also useful for diagnosis especially in those with a low-level of parasitemia and can be used to detect any Babesia DNA levels in blood that show no parasites on blood smear.1 One downfall of PCR is that the DNA detected does not always equate to live parasites and can be detected in blood for up to 26 months post-infection.1
    Indirect immunofluorescence can be used in rare cases with high suspicion of babesiosis and negative blood smears and PCR.1 These cases are seen more in blood donors that have been implicated in transfusion-acquired disease or in asymptomatic patients who have cleared the disease without treatment but may have complications following the infection.1
    The diagnosis should be considered in patients who present with flu-like symptoms and who either live in or have traveled recently to endemic areas. Also, patients with symptom who have been previously diagnosed with Lyme disease, Anaplasmosis, Powassan or other tick-borne illnesses should also be suspected.
    Treatment should be initiated in any symptomatic patients with parasitemia confirmed via blood smear or PCR. If a patient is asymptomatic but has a confirmed diagnosis of babesiosis, treatment is not necessary until symptomatic. Asymptomatic individuals who have persistent infection for > 3months as confirmed by blood smear or PCR should also consider treatment.
    The most effective therapy is the use of antimicrobial agents, specifically atovaquone plus azithromycin or quinine plus clindamycin.3 Both options are equally efficacious but Clindamycin plus quinine is typically reserved for severe cases (requiring hospitalization) or those who are immunocompromised.1 The typical regimen is to take both drugs orally for 7 to 10 days.3 Atovaquone plus azithromycin is the preferred treatment as the combination is better tolerated with less adverse effects and those who cannot tolerate PO can be given IV quinidine and clindamycin.1 Atovaquone is better absorbed when taken with a fatty meal, however azithromycin from capsules have a 50% reduced absorption when taken with food, which is why immediate-release azithromycin suspension is preferred for efficacy.1
    The typical dosing for Atovaquone is 750mg orally every 12 hours and Azithromycin is 500mg/day orally on day 1 and 250mg/day from days 2-10.3 In the case of Clindamycin, 300-600mg is given IV every 6 hours or 600mg orally every 8 hours.3 Quinine is typically dosed at 650mg orally every 6 to 8 hours.3 The dosing is modified for different species of pathogen and for children.
    There are some serious side effects associated with these treatments. Serial ECG’s need to be monitored while on quinines as they can prolong QT intervals and precipitate Toursades de Pointes.3 It is for this reason, that we tailored our treatment toward avoiding this side effect in our patient who had already experienced ventricular arrhythmias during his work-up. Pregnant patients or those breastfeeding should not take quinine or atovaquone as it is a category C drug and leads to abnormal limb development and neurological complications.3
    If patients fail treatment with clindamycin plus quinine, there have been studies that have shown some success with azithromycin plus quinine.3 There also has been success with high doses of clindamycin plus azithromycin and doxycycline especially in immunocompromised patients with recurrent symptoms.1 Other antimalarial agents have been unsuccessful in treating babesiosis and should not be considered for alternative treatment. In cases with high parasitemia and severe hemolytic anemia, blood exchange transfusions should be considered.1

    References
    1. Gelfand, JA and Vannier, EG. Clinical manifestations, diagnosis, treatment, and prevention of babesiosis. https://www.uptodate.com/contents/clinical-manifestations-diagnosis-treatment-and-prevention-of-babesiosis?source=search_result&search=babesiosis&selectedTitle=1~7. Accessed 13 Aug. 2017
    2. CDC – DPDx – Malaria – Diagnostic Findings. https://www.cdc.gov/dpdx/malaria/dx.html. Accessed 13 Aug. 2017
    3. Babesiosis Treatment & Management: Approach Considerations, Pharmacologic Therapy, Exchange Transfusion and Supportive Care. http://emedicine.medscape.com/article/212605-treatment. Accessed 13 Aug. 2017

  5. Question 5 of 8
    5. Question

    An 81-year-old female presents to the emergency department with progressive confusion, disorientation, headache, nausea, and vomiting for the past 24 hours. Physical exam is benign and appears euvolemic. A basic metabolic panel is run and is significant for hyponatremia with a serum sodium of 119 mEq/L. Further testing reveals a decreased serum osmolarity, increased urine osmolarity, and a normal TSH and cortisol level. Of all the following choices which is the most likely diagnosis?

    Correct

    Answer is B. The patient is presenting with signs and symptoms of hyponatremia, specifically from SIADH. The lab diagnostics are valid for SIADH with decreased serum sodium and hypo-osmolality1,3. In addition, the patient is excreting sodium due to having a water problem with ADH and not a true salt deficiency. Therefore, the patient presents with increased urine sodium and increased urine osmolality2,3.
    From the history, one may believe the patient is dehydrated explaining the signs and symptoms of hyponatremia. However, in the setting of dehydration the urine sodium would be decreased, not increased 1. Nephrotic would be an inappropriate answer for this question. The patient’s physical exam is benign. In a patient who is experiencing nephrotic syndrome, they would be edematous and third spacing fluid. In addition, this case does not offer any information pertaining to a UA, which would be needed in the diagnosis of nephrotic syndrome. Patients with nephrotic syndrome present with proteinuria1,4. Psychogenic water intoxication occurs from marked free water intake which results in a dilutional hyponatremia with low serum osmolality similar to that of SIADH1,4. Like SIADH, the patient often maintains euvolemia via renal excretion of sodium. In a patient with psychogenic water intoxication, the urine sodium is often elevated4. However, in this situation there is also an increase in free water excretion, which dilutes the urine and overwhelms the ability of ADH. In psychogenic water intoxication, the urine osmolality would be low unlike SIADH with a high urine osmolality1,4.

    Discussion:
    SIADH is a syndrome in which there is excessive ADH in a patient who is euvolemic. The excess ADH interferes with free water excretion, resulting in an imbalance of input verses output and the devilment of dilutional hyponatremia1,3. Under normal circumstances, the hypothalamus can detect that there is too much water in the blood, and less ADH would be secreted by the pituitary gland into the blood stream1. The kidneys would then absorb less water from the blood in the collecting duct, and dilute urine would be produced which would bring the blood water level back to normal1. In the case of SIADH, the increase in ADH results in increased water reabsorption by the kidneys and decreased aldosterone secretion. SIADH is the most common cause of euvolemic hyponatremia in hospitalized patients3.
    Despite being presenting with hyponatremia, SIADH is not a problem of Na deficiency but of problem with excess water. The ADH increases the water permeability of the renal collection ducts, thereby increasing renal water absorption3. Most of the water absorption is intracellular, but there is also an expansion in the extracellular volume. It is the expansion of the extracellular volume that causes excretion of sodium, and to some degree potassium excretion. This excretion of sodium occurs until a steady state of sodium excreted in the urine matches sodium intake3. The overall result is a dilutional hyponatremia. The most severe risk of SIADH is the brain’s response hypo-osmolality, which can cause brain edema3.
    SIADH can be caused by over secretion of ADH from the hypothalamus or from an ectopically. The etiology can be broken down into four main categories: neoplasia’s, nervous system disorders, pulmonary diseases, and drug induced. Pharmacologic agents that increase AVP release include: isoproterenol, MAOIs, and tricyclic antidepressants1. There are also drugs that can increase the effects of AVP action such as theophylline and some hypoglycemic agents1. Some examples of the other three broad categories include, but are by no means limited to: lung carcinoma, mesothelioma, cerebellar and cerebral atrophy, cerebrovascular accident, multiple sclerosis, asthma, COPD, pneumothorax, and cystic fibrosis3.
    Hyponatremia tends to occur more often in the hospital setting, largely due to the administration of hypotonic IV fluids in the setting of SIADH1,3. It is noted that those who have SIADH experience more hyponatremic events with increasing age over thirty years old3. Men have a higher likely hood of having hyponatremia in the hospital with SIADH than their woman counterpart3. Low body weight is also described as a risk factor for developing hyponatremia in the setting of SIADH3.
    Evaluation of someone presenting with SIADH starts with a history of new medications, changes in fluid intake, and fluid output. The H&P should help identify the etiology of SIADH. The physical exam should help make the diagnosis based on determining the patient’s volume status. Patients with SIADH often present with anorexia, nausea, and vomiting1,2,3,4. In addition, they can develop generalized muscle weakness, myoclonus, tremor, hyporeflexia, ataxia, and dysarthria3. These patients may progress to develop neurological symptoms such as headache, dulled sensorium, confusion, convulsions, and even death due to untreated brain swelling1,2,3.
    Laboratory testing is beneficial in the diagnosis of SIADH. Tests include; but are not limited to: BMP, serum osmolality, urine osmolality, serum uric acid, serum cortisol, and TSH3. In the setting of SIADH, the patient should present with serum hypo-osmolality and hyponatremia with urine hyper osmolality1,2,3. The remainder of the discussed lab values should be within normal reference ranges. Other conditions that alter volume status should be ruled out; such as: CHF, hypovolemia from any cause, and renal insufficiency.
    The rapidity of correction of hyponatremia depends on the degree of the hyponatremia, symptomatic presentation, and acute vs chronic. Extreme hyponatremia and an inappropriate approach to its treatment can both have disastrous consequences and often a nephrology consult is needed. Correcting hyponatremia too rapidly may result in central pontine myelinolysis with permanent neurologic defects1, 2. In order to avoid neurologic defects, it is advised to raise serum sodium by 0.5-1mEq/hr until a serum sodium of 125-130mEq/L is obtained2, 3, 4. However, in an acute setting with hyponatremia occurring within 48 hours 3% hypertonic saline can be used at 513mEq/L. In this situation loop diuretics with saline, vasopressin-2 receptor antagonists, and water restriction can also be effective3. In the more chronic, asymptomatic setting fluid restriction with or without vasopressin-2 receptor antagonists can be used2, 3.

    References:
    1.)Papadakis MA, McPhee SJ. Current Medical Diagnosis & Treamtent. 54th ed. Cenveo
    Publisher Services; 2015.

    2.)Pillai BP, Unnikrishnan AG, Pavithran PV. Syndrome of inappropriate antidiuretic hormone
    secretion: Revisiting a classical endocrine disorder. Indian Journal of Endocrinology and
    Metabolism. 2011;15(Suppl3):S208-S215. doi:10.4103/2230-8210.84870.

    3.)Thomas CP. Syndrome of Inappropriate Antidiuretic Hormone Secretion Workup. Syndrome
    of Inappropriate Antidiuretic Hormone Secretion Workup: Approach Considerations,
    Laboratory Tests, Volume Assessment. http://emedicine.medscape.com/article/246650- workup#c7. Published July 17, 2017. Accessed August 12, 2017.

    4.)Wiener, Longo, Fauci, et al. Harrison’s Principles of Internal Medicine. 18th ed. Cenveo
    Publisher Services; 2012.

    Incorrect

    Answer is B. The patient is presenting with signs and symptoms of hyponatremia, specifically from SIADH. The lab diagnostics are valid for SIADH with decreased serum sodium and hypo-osmolality1,3. In addition, the patient is excreting sodium due to having a water problem with ADH and not a true salt deficiency. Therefore, the patient presents with increased urine sodium and increased urine osmolality2,3.
    From the history, one may believe the patient is dehydrated explaining the signs and symptoms of hyponatremia. However, in the setting of dehydration the urine sodium would be decreased, not increased 1. Nephrotic would be an inappropriate answer for this question. The patient’s physical exam is benign. In a patient who is experiencing nephrotic syndrome, they would be edematous and third spacing fluid. In addition, this case does not offer any information pertaining to a UA, which would be needed in the diagnosis of nephrotic syndrome. Patients with nephrotic syndrome present with proteinuria1,4. Psychogenic water intoxication occurs from marked free water intake which results in a dilutional hyponatremia with low serum osmolality similar to that of SIADH1,4. Like SIADH, the patient often maintains euvolemia via renal excretion of sodium. In a patient with psychogenic water intoxication, the urine sodium is often elevated4. However, in this situation there is also an increase in free water excretion, which dilutes the urine and overwhelms the ability of ADH. In psychogenic water intoxication, the urine osmolality would be low unlike SIADH with a high urine osmolality1,4.

    Discussion:
    SIADH is a syndrome in which there is excessive ADH in a patient who is euvolemic. The excess ADH interferes with free water excretion, resulting in an imbalance of input verses output and the devilment of dilutional hyponatremia1,3. Under normal circumstances, the hypothalamus can detect that there is too much water in the blood, and less ADH would be secreted by the pituitary gland into the blood stream1. The kidneys would then absorb less water from the blood in the collecting duct, and dilute urine would be produced which would bring the blood water level back to normal1. In the case of SIADH, the increase in ADH results in increased water reabsorption by the kidneys and decreased aldosterone secretion. SIADH is the most common cause of euvolemic hyponatremia in hospitalized patients3.
    Despite being presenting with hyponatremia, SIADH is not a problem of Na deficiency but of problem with excess water. The ADH increases the water permeability of the renal collection ducts, thereby increasing renal water absorption3. Most of the water absorption is intracellular, but there is also an expansion in the extracellular volume. It is the expansion of the extracellular volume that causes excretion of sodium, and to some degree potassium excretion. This excretion of sodium occurs until a steady state of sodium excreted in the urine matches sodium intake3. The overall result is a dilutional hyponatremia. The most severe risk of SIADH is the brain’s response hypo-osmolality, which can cause brain edema3.
    SIADH can be caused by over secretion of ADH from the hypothalamus or from an ectopically. The etiology can be broken down into four main categories: neoplasia’s, nervous system disorders, pulmonary diseases, and drug induced. Pharmacologic agents that increase AVP release include: isoproterenol, MAOIs, and tricyclic antidepressants1. There are also drugs that can increase the effects of AVP action such as theophylline and some hypoglycemic agents1. Some examples of the other three broad categories include, but are by no means limited to: lung carcinoma, mesothelioma, cerebellar and cerebral atrophy, cerebrovascular accident, multiple sclerosis, asthma, COPD, pneumothorax, and cystic fibrosis3.
    Hyponatremia tends to occur more often in the hospital setting, largely due to the administration of hypotonic IV fluids in the setting of SIADH1,3. It is noted that those who have SIADH experience more hyponatremic events with increasing age over thirty years old3. Men have a higher likely hood of having hyponatremia in the hospital with SIADH than their woman counterpart3. Low body weight is also described as a risk factor for developing hyponatremia in the setting of SIADH3.
    Evaluation of someone presenting with SIADH starts with a history of new medications, changes in fluid intake, and fluid output. The H&P should help identify the etiology of SIADH. The physical exam should help make the diagnosis based on determining the patient’s volume status. Patients with SIADH often present with anorexia, nausea, and vomiting1,2,3,4. In addition, they can develop generalized muscle weakness, myoclonus, tremor, hyporeflexia, ataxia, and dysarthria3. These patients may progress to develop neurological symptoms such as headache, dulled sensorium, confusion, convulsions, and even death due to untreated brain swelling1,2,3.
    Laboratory testing is beneficial in the diagnosis of SIADH. Tests include; but are not limited to: BMP, serum osmolality, urine osmolality, serum uric acid, serum cortisol, and TSH3. In the setting of SIADH, the patient should present with serum hypo-osmolality and hyponatremia with urine hyper osmolality1,2,3. The remainder of the discussed lab values should be within normal reference ranges. Other conditions that alter volume status should be ruled out; such as: CHF, hypovolemia from any cause, and renal insufficiency.
    The rapidity of correction of hyponatremia depends on the degree of the hyponatremia, symptomatic presentation, and acute vs chronic. Extreme hyponatremia and an inappropriate approach to its treatment can both have disastrous consequences and often a nephrology consult is needed. Correcting hyponatremia too rapidly may result in central pontine myelinolysis with permanent neurologic defects1, 2. In order to avoid neurologic defects, it is advised to raise serum sodium by 0.5-1mEq/hr until a serum sodium of 125-130mEq/L is obtained2, 3, 4. However, in an acute setting with hyponatremia occurring within 48 hours 3% hypertonic saline can be used at 513mEq/L. In this situation loop diuretics with saline, vasopressin-2 receptor antagonists, and water restriction can also be effective3. In the more chronic, asymptomatic setting fluid restriction with or without vasopressin-2 receptor antagonists can be used2, 3.

    References:
    1.)Papadakis MA, McPhee SJ. Current Medical Diagnosis & Treamtent. 54th ed. Cenveo
    Publisher Services; 2015.

    2.)Pillai BP, Unnikrishnan AG, Pavithran PV. Syndrome of inappropriate antidiuretic hormone
    secretion: Revisiting a classical endocrine disorder. Indian Journal of Endocrinology and
    Metabolism. 2011;15(Suppl3):S208-S215. doi:10.4103/2230-8210.84870.

    3.)Thomas CP. Syndrome of Inappropriate Antidiuretic Hormone Secretion Workup. Syndrome
    of Inappropriate Antidiuretic Hormone Secretion Workup: Approach Considerations,
    Laboratory Tests, Volume Assessment. http://emedicine.medscape.com/article/246650- workup#c7. Published July 17, 2017. Accessed August 12, 2017.

    4.)Wiener, Longo, Fauci, et al. Harrison’s Principles of Internal Medicine. 18th ed. Cenveo
    Publisher Services; 2012.

  6. Question 6 of 8
    6. Question

    A 62 year old female is brought to your office by her family who is concerned due to her sudden inability to move the right side of her face. The patient’s family reports they were eating breakfast when they noticed that she was having difficulty chewing her food, and the right side of her face was drooping. The patient notes that her right eye has been dry, and when she woke up this morning she noticed she could no longer close it. On examination you notice right sided facial paralysis which includes sagging, loss of the nasolabial fold, inability to close the right eye, and inability to raise the forehead on the right side. She denies any visual changes, or pain. The remainder of her neurological exam is unremarkable. What is the most likely cause of this patient’s condition?

    Correct

    Case Discussion
    Bell’s Palsy (BP), which may also be known as Idiopathic Facial Paralysis (IFP), is a disorder involving cranial nerve VII.1,2 It is described as the most common cause of sudden onset unilateral facial paralysis.1,3 When BP was first diagnosed in the 1800’s the etiology was thought to be trauma to the peripheral nerve branches, but since this theory has been dispelled.1 The most common cause of BP is herpes simplex virus (HSV). There are many other, less common, etiologies such as Lyme disease, herpes zoster, syphilis, Epstein Barr virus, cytomegalovirus, HIV, inflammation, or microvascular disease.3Another hypothesis is that BP can have a genetic etiology; however, since the most common causes of BP are viral, and therefore contagious, the true etiology may not be genetic, but instead infectious in nature, and merely passed onto family members through contact.3
    BP accounts for over half of all cases of facial nerve palsy.1,3The incidence of BP is approximately 23/100,000 in the United States.3 Although there has been no link found between race, geographic location, or sex there is a slight increase of cases in females between 10-19 years old compared to males of the same age group.2,3 BP rarely occurs before age ten, and most commonly occurs after age 60.2,3 A major risk factor for BP is pregnancy, with a 3.3 times higher risk of contracting BP than nonpregnant women, with the highest rate occurring in the third trimester, and first week postpartum.1,3 Other risk factors include diabetes, obesity, and hypertension.2
    BP is a sudden onset of peripheral facial palsy of which there may be many causes, while the pathophysiology is widely debated there are a number of hypothesis. One hypothesis states that HSV causes BP when the virus is spread along the axon, and then the reactivated virus is multiplied causing inflammation, demyelination and palsy of cranial nerve VII (facial nerve).1 Another hypothesis states that the palsy is caused when there is edema and ischemia of the facial nerve which leads to compression of the nerve in the bony canal. 3 However, this theory states that the cause of the edema and ischemia are unknown. BP is called a peripheral nerve palsy because the injury to the nerve occurs peripherally compared to the nerve’s nucleus. If the injury occurs proximal to the geniculate ganglion, there is motor paralysis, gustatory, and autonomic abnormalities seen. If the lesion is between the geniculate ganglion and the chorda tympani then lacrimation is spared. Finally, if the lesion is at the stylomastoid foramen this results solely in motor abnormalities. 3
    Thorough history and physical examination should be obtained. Patients with BP usually present with an acute onset of unilateral facial paralysis that has occurred over several hours.1 Patients may complain of decreased tearing on the affected side, hyperacusis, loss of taste on the anterior 2/3 of the tongue, inability to smile, otalgia, and/or posterior auricular pain.1,3 Physical examination should include a full general and neurological examination including assessment of all cranial nerves, sensory, motor, and cerebellar testing.3 On exam you will notice inability to close the eye on the affected side along with, eyebrow dropping, facial sagging, and loss of the nasolabial fold.1 It is important to note if the forehead muscles are spared or not in order to differentiate between a central lesion or a peripheral lesion. The muscles will be spared in a central lesion. 1 All other neurological abnormalities that cannot be explained by a facial nerve palsy should be referred for further testing, such as MRI, lumbar puncture, or EMG. 3
    The diagnosis of BP is one of exclusion that can usually be made if there has been a thorough history and physical examination done. Imaging is not normally needed for patients with a typical facial nerve palsy presentation; however, as stated above, further studies and imaging should be ordered for patients who do not present in a traditional fashion.1 Serological testing for Lyme disease, HSV, etc. may be done in order to determine the etiology of the palsy.1 Additional testing may include a CBC with differential to look for signs of infection, and screening for diabetes mellitus.2
    Treatment of BP of suspected viral etiology is with oral glucocorticoids, and in some cases the addition of an antiviral. Treatment should be started within three days of the onset for the best outcomes. The guidelines suggest Prednisone 60-80mg/day for one week as the glucocorticoid of choice. For severe cases addition of an antiviral, such as Valacyclovir 1000mg three times a day for one week is recommended. BP due to other etiologies such as Lyme disease should be treated based on the cause. For patients that are unable to close their eye, eye care is of utmost importance, due to the cornea being at increased risk for damage. Using artificial tears, or other lubricating eye drops along with protective patch or goggles will decrease risk of damage. Most patients start to see improvement within the first 21 days, however some patients may have long term sequel. Education about the long term effects should be discussed with the patient.1 Referral for further investigation should be made if the palsy does not show improvements in three weeks.2

    References
    1. Ronthal M. Bell’s palsy: Pathogenesis, clinical features, and diagnosis in adults. Bell’s palsy: Pathogenesis, clinical features, and diagnosis in adults. https://www.uptodate.com/contents/bells-palsy-pathogenesis-clinical-features-and-diagnosis-in-adults?source=see_link#H69095511. Published December 1, 2016. Accessed August 8, 2017.
    2. Patel D, Levin K. Bell palsy: Clinical examination and management. MDedge. http://www.mdedge.com/ccjm/article/100703/neurology/bell-palsy-clinical-examination-and-management. Published July 2015. Accessed August 8, 2017.
    3. Taylor D. Bell Palsy Clinical Presentation. Bell Palsy Clinical Presentation: History, Physical Examination, Grading. http://emedicine.medscape.com/article/1146903-clinical#b3. Published July 19, 2017. Accessed August 8, 2017.

    Incorrect

    Case Discussion
    Bell’s Palsy (BP), which may also be known as Idiopathic Facial Paralysis (IFP), is a disorder involving cranial nerve VII.1,2 It is described as the most common cause of sudden onset unilateral facial paralysis.1,3 When BP was first diagnosed in the 1800’s the etiology was thought to be trauma to the peripheral nerve branches, but since this theory has been dispelled.1 The most common cause of BP is herpes simplex virus (HSV). There are many other, less common, etiologies such as Lyme disease, herpes zoster, syphilis, Epstein Barr virus, cytomegalovirus, HIV, inflammation, or microvascular disease.3Another hypothesis is that BP can have a genetic etiology; however, since the most common causes of BP are viral, and therefore contagious, the true etiology may not be genetic, but instead infectious in nature, and merely passed onto family members through contact.3
    BP accounts for over half of all cases of facial nerve palsy.1,3The incidence of BP is approximately 23/100,000 in the United States.3 Although there has been no link found between race, geographic location, or sex there is a slight increase of cases in females between 10-19 years old compared to males of the same age group.2,3 BP rarely occurs before age ten, and most commonly occurs after age 60.2,3 A major risk factor for BP is pregnancy, with a 3.3 times higher risk of contracting BP than nonpregnant women, with the highest rate occurring in the third trimester, and first week postpartum.1,3 Other risk factors include diabetes, obesity, and hypertension.2
    BP is a sudden onset of peripheral facial palsy of which there may be many causes, while the pathophysiology is widely debated there are a number of hypothesis. One hypothesis states that HSV causes BP when the virus is spread along the axon, and then the reactivated virus is multiplied causing inflammation, demyelination and palsy of cranial nerve VII (facial nerve).1 Another hypothesis states that the palsy is caused when there is edema and ischemia of the facial nerve which leads to compression of the nerve in the bony canal. 3 However, this theory states that the cause of the edema and ischemia are unknown. BP is called a peripheral nerve palsy because the injury to the nerve occurs peripherally compared to the nerve’s nucleus. If the injury occurs proximal to the geniculate ganglion, there is motor paralysis, gustatory, and autonomic abnormalities seen. If the lesion is between the geniculate ganglion and the chorda tympani then lacrimation is spared. Finally, if the lesion is at the stylomastoid foramen this results solely in motor abnormalities. 3
    Thorough history and physical examination should be obtained. Patients with BP usually present with an acute onset of unilateral facial paralysis that has occurred over several hours.1 Patients may complain of decreased tearing on the affected side, hyperacusis, loss of taste on the anterior 2/3 of the tongue, inability to smile, otalgia, and/or posterior auricular pain.1,3 Physical examination should include a full general and neurological examination including assessment of all cranial nerves, sensory, motor, and cerebellar testing.3 On exam you will notice inability to close the eye on the affected side along with, eyebrow dropping, facial sagging, and loss of the nasolabial fold.1 It is important to note if the forehead muscles are spared or not in order to differentiate between a central lesion or a peripheral lesion. The muscles will be spared in a central lesion. 1 All other neurological abnormalities that cannot be explained by a facial nerve palsy should be referred for further testing, such as MRI, lumbar puncture, or EMG. 3
    The diagnosis of BP is one of exclusion that can usually be made if there has been a thorough history and physical examination done. Imaging is not normally needed for patients with a typical facial nerve palsy presentation; however, as stated above, further studies and imaging should be ordered for patients who do not present in a traditional fashion.1 Serological testing for Lyme disease, HSV, etc. may be done in order to determine the etiology of the palsy.1 Additional testing may include a CBC with differential to look for signs of infection, and screening for diabetes mellitus.2
    Treatment of BP of suspected viral etiology is with oral glucocorticoids, and in some cases the addition of an antiviral. Treatment should be started within three days of the onset for the best outcomes. The guidelines suggest Prednisone 60-80mg/day for one week as the glucocorticoid of choice. For severe cases addition of an antiviral, such as Valacyclovir 1000mg three times a day for one week is recommended. BP due to other etiologies such as Lyme disease should be treated based on the cause. For patients that are unable to close their eye, eye care is of utmost importance, due to the cornea being at increased risk for damage. Using artificial tears, or other lubricating eye drops along with protective patch or goggles will decrease risk of damage. Most patients start to see improvement within the first 21 days, however some patients may have long term sequel. Education about the long term effects should be discussed with the patient.1 Referral for further investigation should be made if the palsy does not show improvements in three weeks.2

    References
    1. Ronthal M. Bell’s palsy: Pathogenesis, clinical features, and diagnosis in adults. Bell’s palsy: Pathogenesis, clinical features, and diagnosis in adults. https://www.uptodate.com/contents/bells-palsy-pathogenesis-clinical-features-and-diagnosis-in-adults?source=see_link#H69095511. Published December 1, 2016. Accessed August 8, 2017.
    2. Patel D, Levin K. Bell palsy: Clinical examination and management. MDedge. http://www.mdedge.com/ccjm/article/100703/neurology/bell-palsy-clinical-examination-and-management. Published July 2015. Accessed August 8, 2017.
    3. Taylor D. Bell Palsy Clinical Presentation. Bell Palsy Clinical Presentation: History, Physical Examination, Grading. http://emedicine.medscape.com/article/1146903-clinical#b3. Published July 19, 2017. Accessed August 8, 2017.

  7. Question 7 of 8
    7. Question

    A 63 year-old female presents to the office with left knee pain for several months. The pain is made worse with stairs and seems to be really achy at the end of the day. She denies prior injury. AP, lateral, and merchant view x-rays show mild joint space narrowing, subchondral sclerosis in the medial compartment, and significant osteophytes in the patella femoral joint (Figures 1, 2, and 3). Based on the Kellgren-Lawrence grading system for knee osteoarthritis, which grade of arthritis does this patient have?

    Correct

    Answer C. The Kellgren-Lawrence (KL) grading system is the most widely accepted assessment tool for quantifying the degree of knee osteoarthritis found on x-ray. Knowing this grading system is of increasing importance today as some insurance carriers are requiring documentation of KL grade 2 or higher knee osteoarthritis on x-ray in order to approve certain treatments like viscosupplementation injections. Viscosupplementation, or hyaluronic acid injections, are gel like injections for the knee that provide lubrication and pain relief for patients with knee arthritis.
    KL grade 1 changes include no obvious joint space narrowing with small osteophytes present, grade 2 changes include definite osteophytes with possible joint space narrowing, grade 3 changes include multiple osteophytes, moderate joint space narrowing, subchondral sclerosis, and possible bony deformity, and grade 4 changes include large osteophytes, severe joint space narrowing, subchondral sclerorsis, and definite bony deformity.
    There are several flaws in the KL grading system, however. The KL system relies on the formation of osteophytes occurring before joint space narrowing, although generally the opposite occurs. Also, several studies have looked at arthroscopic findings in patients with early articular degeneration compared to KL grade and concluded that the KL system can significantly underestimate the degree of osteoarthritis in the knee.
    References
    1. Kijowski R, Blankenbaker D, Stanton P, Fine J, De Smet A. Arthroscopic Validation of Radiographic Grading Scales of Osteoarthritis of the Tibiofemoral Joint. AJR. 2006. 187. 794-799.
    2. Kellgren-Lawrence (KL) grading system. https://radiopaedia.org. Accessed on 7/23/17.

    Incorrect

    Answer C. The Kellgren-Lawrence (KL) grading system is the most widely accepted assessment tool for quantifying the degree of knee osteoarthritis found on x-ray. Knowing this grading system is of increasing importance today as some insurance carriers are requiring documentation of KL grade 2 or higher knee osteoarthritis on x-ray in order to approve certain treatments like viscosupplementation injections. Viscosupplementation, or hyaluronic acid injections, are gel like injections for the knee that provide lubrication and pain relief for patients with knee arthritis.
    KL grade 1 changes include no obvious joint space narrowing with small osteophytes present, grade 2 changes include definite osteophytes with possible joint space narrowing, grade 3 changes include multiple osteophytes, moderate joint space narrowing, subchondral sclerosis, and possible bony deformity, and grade 4 changes include large osteophytes, severe joint space narrowing, subchondral sclerorsis, and definite bony deformity.
    There are several flaws in the KL grading system, however. The KL system relies on the formation of osteophytes occurring before joint space narrowing, although generally the opposite occurs. Also, several studies have looked at arthroscopic findings in patients with early articular degeneration compared to KL grade and concluded that the KL system can significantly underestimate the degree of osteoarthritis in the knee.
    References
    1. Kijowski R, Blankenbaker D, Stanton P, Fine J, De Smet A. Arthroscopic Validation of Radiographic Grading Scales of Osteoarthritis of the Tibiofemoral Joint. AJR. 2006. 187. 794-799.
    2. Kellgren-Lawrence (KL) grading system. https://radiopaedia.org. Accessed on 7/23/17.

  8. Question 8 of 8
    8. Question

    A 56 year-old female presents to your office with a several month history of left forefoot pain. She denies prior injury or known precipitating event. The pain seems to be located between the 2nd and 3rd toes and is made worse with tight shoes. She really notices the pain when she wears high heels and works out at the gym. On exam you note increased pain when squeezing the metatarsal heads together. AP and lateral x-rays of the left foot are shown in figures 1 and 2. Which choice below is commonly used to confirm the patient’s diagnosis?

    Correct

    Answer C. Interdigital neuritis, often referred to as Morton’s neuroma, is a compressive neuropathy that most commonly involves the 3rd interdigital nerve between the 3rd and 4th metatarsal heads. The condition is most prevalent in middle aged women. The pathophysiology of the condition is poorly understood but many believe it is due to mechanical trauma, entrapment, and/or tethering of the nerve. Symptoms include burning and paresthesia’s in the involved toes that are made worse with narrow toe box shoes or high heels. Patients may feel like they are walking on a small pebble in their shoe. The pain is often relieved with removing the shoe and taking a rest from walking. On exam there is generally no obvious deformity of the foot and pain can be palpated at the involved interspace between the metatarsal heads. Pain is often made worse by compressing the metatarsal heads together. 1,2
    The diagnosis of Morton’s neuroma is often made with history and physical alone. If there is any doubt in the diagnosis, 1 ml of lidocaine can be injected into the dorsal intermetatarsal ligament between the metatarsal heads. Temporary relief of pain after the injection confirms the diagnosis. MRI and ultrasound can identify an interdigital neuroma, but are rarely used due to the diagnostic accuracy of history and physical alone. Conservative treatment include NSAIDS, shoes with wide toe boxes, and metatarsal pads to push the metatarsal heads away from each other. Steroid injections may also provide short term relief of pain. Morton’s neuroma is generally treated surgically as only 20% of patients will get better with conservative treatment. The standard surgical treatment involves resection of the interdigital nerve (neurectomy). 1,2

    References
    1. Hidalgo-Ovejero, AM; Martinez-Grande M, Sanchez-Villares JJ, Garcia-Mata S, Lasanta P, Coughlin MJ, Pinsonneault T. Clinical Examination and Imaging Studies in the Diagnosis of Interdigital Neuroma. Journal of Bone & Joint Surgery – American Volume: July 2002 – Volume 84 – Issue 7 – p 1276–1278.
    2. Interdigital Neuroma. http://www.orthobullets.com. Accessed on 4/26/17.

    Incorrect

    Answer C. Interdigital neuritis, often referred to as Morton’s neuroma, is a compressive neuropathy that most commonly involves the 3rd interdigital nerve between the 3rd and 4th metatarsal heads. The condition is most prevalent in middle aged women. The pathophysiology of the condition is poorly understood but many believe it is due to mechanical trauma, entrapment, and/or tethering of the nerve. Symptoms include burning and paresthesia’s in the involved toes that are made worse with narrow toe box shoes or high heels. Patients may feel like they are walking on a small pebble in their shoe. The pain is often relieved with removing the shoe and taking a rest from walking. On exam there is generally no obvious deformity of the foot and pain can be palpated at the involved interspace between the metatarsal heads. Pain is often made worse by compressing the metatarsal heads together. 1,2
    The diagnosis of Morton’s neuroma is often made with history and physical alone. If there is any doubt in the diagnosis, 1 ml of lidocaine can be injected into the dorsal intermetatarsal ligament between the metatarsal heads. Temporary relief of pain after the injection confirms the diagnosis. MRI and ultrasound can identify an interdigital neuroma, but are rarely used due to the diagnostic accuracy of history and physical alone. Conservative treatment include NSAIDS, shoes with wide toe boxes, and metatarsal pads to push the metatarsal heads away from each other. Steroid injections may also provide short term relief of pain. Morton’s neuroma is generally treated surgically as only 20% of patients will get better with conservative treatment. The standard surgical treatment involves resection of the interdigital nerve (neurectomy). 1,2

    References
    1. Hidalgo-Ovejero, AM; Martinez-Grande M, Sanchez-Villares JJ, Garcia-Mata S, Lasanta P, Coughlin MJ, Pinsonneault T. Clinical Examination and Imaging Studies in the Diagnosis of Interdigital Neuroma. Journal of Bone & Joint Surgery – American Volume: July 2002 – Volume 84 – Issue 7 – p 1276–1278.
    2. Interdigital Neuroma. http://www.orthobullets.com. Accessed on 4/26/17.

Proudly powered by WordPress | Theme: Motif by WordPress.com.